Limit of Sequence: "2, $\frac{3}{2}, \frac{4}{3}, \frac{5}{4}$

  • Thread starter Thread starter NoLimits
  • Start date Start date
  • Tags Tags
    Limit Sequence
Click For Summary
The discussion revolves around finding the nth term and limit of the sequence 2, 3/2, 4/3, 5/4. The initial approach suggested using L'Hôpital's rule, but there was confusion about determining a limit from a sequence of changing numbers. It was clarified that the limit can be found by rewriting the expression as (n+1)/n = 1 + 1/n and then taking the limit as n approaches infinity. The limit of the sequence is confirmed to be 1, as the additional term 1/n approaches zero. Understanding the cancellation of terms is essential for accurately determining the limit.
NoLimits
Messages
10
Reaction score
0
Hello again,

I am having trouble with a particular limit problem and would appreciate any help/pointers you can offer. The question is asking for the nth term of the sequence 2, \frac{3}{2}, \frac{4}{3}, \frac{5}{4}

.. and also asks for a limit of the sequence. My immediate guess was to apply l'hopital's rule, which would mean setting n to approach infinity and using something like this:

lim_n→∞ \frac{n+1}{n}

It seems to me like it could work, however I do not understand how an actual 'limit' value can be determined from a sequence of unknown and changing numbers ('n'). What I mean is, in order to make my limit work then the nth term would have to equal infinity, would it not?

** Edit **: According to an online limit solver the limit is 1, which I can see is possible if the n values are canceled out.
 
Physics news on Phys.org
NoLimits said:
Hello again,

I am having trouble with a particular limit problem and would appreciate any help/pointers you can offer. The question is asking for the nth term of the sequence 2, \frac{3}{2}, \frac{4}{3}, \frac{5}{4}

.. and also asks for a limit of the sequence. My immediate guess was to apply l'hopital's rule, which would mean setting n to approach infinity and using something like this:

lim_n→∞ \frac{n+1}{n}

It seems to me like it could work, however I do not understand how an actual 'limit' value can be determined from a sequence of unknown and changing numbers ('n'). What I mean is, in order to make my limit work then the nth term would have to equal infinity, would it not?

** Edit **: According to an online limit solver the limit is 1, which I can see is possible if the n values are canceled out.

You don't "cancel out" the ##n##'s. You write as$$
\frac{n+1} n = \frac n n + \frac 1 n = 1 + \frac 1 n$$and take the limit as ##n\to\infty## of that.
 
  • Like
Likes 1 person
Question: A clock's minute hand has length 4 and its hour hand has length 3. What is the distance between the tips at the moment when it is increasing most rapidly?(Putnam Exam Question) Answer: Making assumption that both the hands moves at constant angular velocities, the answer is ## \sqrt{7} .## But don't you think this assumption is somewhat doubtful and wrong?

Similar threads

  • · Replies 1 ·
Replies
1
Views
2K
  • · Replies 12 ·
Replies
12
Views
2K
  • · Replies 4 ·
Replies
4
Views
2K
  • · Replies 9 ·
Replies
9
Views
3K
  • · Replies 1 ·
Replies
1
Views
1K
  • · Replies 4 ·
Replies
4
Views
2K
  • · Replies 8 ·
Replies
8
Views
2K
  • · Replies 2 ·
Replies
2
Views
2K
  • · Replies 7 ·
Replies
7
Views
2K
  • · Replies 9 ·
Replies
9
Views
2K